2
$\begingroup$

Question. Let sequence of functions $f_{n}(x)$ is uniformly bounded on $[0,\infty)$,and $\{f_{n}\}$ converges to $f(x)$ uniformly on each compact subset of $\mathbb R^{+}$,and fixed to $n$,then $f_{n}(x)$ is increasing or decreasing functions on $[0,\infty)$,and $g(x)$ is Improper integrable on $[0,\infty)$.

Show that: $$ f(x)g(x),\,\,f_{n}(x)g(x), \,\,n\in\mathbb N, $$ are all Improper integrable on $[0,\infty)$ and $$\lim_{n\to\infty}\int_{0}^{\infty}f_{n}(x)g(x)\,dx=\int_{0}^{\infty}f(x)g(x)\,dx$$

My try: Since Improper integrable is :http://en.wikipedia.org/wiki/Improper_integral

Since $$ \lvert f_{n}(x)\rvert\le M,\quad x\ge 0, $$ and since $f_{n}(x)$ converges to $f(x)$ uniformly on each compact subset of $R^{+}$, then we have $$\int_{-A}^{A}\lvert f_{n}(x)-f(x)\rvert \,dx\le\dfrac{\varepsilon}{3}$$ Then I can't,and this problem is from Mathematics examination question

Thank you

$\endgroup$
2
  • $\begingroup$ why close it? Thank you $\endgroup$
    – math110
    Dec 27, 2013 at 8:13
  • $\begingroup$ It is indeed unclear what you are asking (that is why someone voted to close). I also can not understand what is stated about functions $f_n$? Are they alltogether increasing or alltogether decreasing or it is just known that for each $f_n$ is decreasing or increasing? $\endgroup$
    – Norbert
    Dec 29, 2013 at 21:36

1 Answer 1

5
+50
$\begingroup$

Clearly, the improper integral of the function $h: [0,\infty)\to\mathbb R$ exists in $\mathbb R$ if and only if $$ \lim_{M,N\to\infty}\int_M^N h(x)\,dx=0. $$

We shall need the following result:

Lemma. If $F,G :[a,b]\to\mathbb R$ functions, with $G$ Riemann integrable and $F$ monotone, then $$ \Big| \int_a^b F(x)\,G(x)\,dx\,\Big| \le \big(|F(b)|+|F(b)-F(a)|\big) \cdot\max_{x\in[a,b]}\Big| \int_a^x G(t)\,dt\,\Big|. $$

Proof. Let $n\in\mathbb N$, and define the partition of $[a,b]$: $$ \tau_j= a+j\frac{b-a}{n}, \quad j=0,\ldots,n. $$ Next we define $F_n: [a,b]\to\mathbb R,$ as a step function, to be equal to $F(a)$, in the interval $[a,\tau_j)$, $F(\tau_1)$, in the interval $[\tau_1,\tau_2)\ldots$ and $F(\tau_{n-1})$, in the interval $[\tau_{n-1},b]$. Then \begin{align} \int_a^b F_n(x)\,G(x)\,dx &=\sum_{k=1}^n\int_{\tau_{k-1}}^{\tau_k} F_n(x)\,G(x)\,dx \\ &=\sum_{k=1}^n F(\tau_{k-1})\int_{\tau_{k-1}}^{\tau_k} G(x)\,dx \\ &\cdots \\&=F(b)\int_a^b g(x)\,dx+\sum_{k=1}^{n-1} \big(F(\tau_{k-1})-F(\tau_k)\big)\int_a^{\tau_k} g(x)\,dx. \end{align} and thus $$ \Big| \int_a^b F_n(x)\,G(x)\,dx\,\Big| \le \big(|F(b)|+|F(b)-F(a)|\big) \cdot\max_{x\in[a,b]}\Big| \int_a^x G(t)\,dt\,\Big|, \quad\text{for all}\,\,n\in\mathbb N, $$ and the proof of the lemma follows from the fact that $$ \Big| \int_a^b \big(F_n(x)\,G(x)-F(x)\,G(x)\big)\,dx\,\Big| \le \sup_{x\in [a,b]} |G(x)|\cdot \Big| \int_a^b \big(F_n(x)-F(x)\big)\,dx\,\Big|, $$ and clearly $\int_a^b F_n(x)\,dx$ is the lower Riemann sum (or upper sum) corresponding to Riemann integral $\int_a^b F(x)\,dx$, and hence $$ \Big| \int_a^b \big(F_n(x)-F(x)\big)\,dx\,\Big|\to 0\quad\text{as}\quad n\to\infty. \tag*{$\Box$} $$

Assume now that $|f_n(x)|, |f(x)|\le K$.

First. For every $n$ the improper integral of the function $f_ng$ exists in $\mathbb R$, as due to the lemma \begin{align} \Big|\int_M^N f_n(x)\,g(x)\,dx\,\Big|&\le \big(|f_n(N)|+ |f_n(M)-f_n(N)|\big)\cdot\max_{x\in[M,N]}\Big| \int_M^N g(t)\,dt\,\Big| \\ &\le 3K \cdot\max_{x\in[M,N]}\Big| \int_M^N g(t)\,dt\,\Big|. \end{align} Clearly, the right hand side tends to zero, as $M,N\to\infty$, and hence $\lim_{M,N\to\infty}\Big|\int_M^N f_n(x)\,g(x)\,dx\,\Big|=0$, which means that the improper integral of the function $f_ng$ exists in $\mathbb R$.

Second. Let $\varepsilon>0$ and $M>0$, such that $\sup_{x\ge M}\Big|\int_M^x g(t)\,dx\,\Big|<\frac{\varepsilon}{12K+1}$, and $n_0\in\mathbb N$, such that $$ |f_n(x)g(x)-f(x)g(x)|<\frac{\varepsilon}{2M},\quad\text{for all}\,\,x\in[0,M]\,\,\text{and}\,\,\, n\ge n_0. $$ Then \begin{align} &\Big|\int_0^\infty f_n(x)\,g(x)\,dx-\int_0^\infty f(x)\,g(x)\,dx\,\Big| \\ &\le \Big|\int_0^M f_n(x)\,g(x)\,dx-\int_0^M f(x)\,g(x)\,dx\,\Big|+ \Big|\int_M^\infty f_n(x)\,g(x)\,dx-\int_M^\infty f(x)\,g(x)\,dx\,\Big| \\ &\le\int_0^M \big|f_n(x)\,g(x)-f(x)\,g(x)\big|\,dx+ \Big|\int_M^\infty f_n(x)\,g(x)\,dx\Big|+\Big|\int_M^\infty f(x)\,g(x)\,dx\,\Big| \\ &\le M\cdot\frac{\varepsilon}{2M}+2\cdot 3K\cdot \frac{\varepsilon}{12K+1}<\varepsilon, \end{align} as $$ \Big|\int_M^\infty f(x)\,g(x)\,dx\Big|\le \big(\sup_{x\ge M}|f(x)|+ \sup_{x\ge M} |f(M)-f(x)| \big)\cdot \sup_{x\ge M}\Big|\int_M^x g(t)\,dx\,\Big|\le 3K\cdot\frac{\varepsilon}{12K+1}. $$

$\endgroup$

You must log in to answer this question.

Not the answer you're looking for? Browse other questions tagged .